0 Daumen
1,1k Aufrufe

Aufgabe:

Gegeben seien die folgenden Basen des Vektorraums \(V = \{p \in R[t] \mid \operatorname{deg}(p) \leq 3\}\) aller Polynome vom Grad
kleiner gleich 3:
$$\mathcal{A} = (t^3, t^2, t, 1)$$
a) Stellen Sie das Polynom \(p(t) = 3t^3 + t^2 − 1\) in Koordinatendarstellung bzgl. Basis \(\mathcal{A}\) dar.


Problem/Ansatz:

\(\lambda_1 t^3+\lambda_2 t^2+\lambda_3 t^1+\lambda 4 t^0 = 3t^3+t^2-1t^0\implies \lambda_1=3,\; \lambda_2=1,\;\lambda_3=0,\;\lambda_4=-1\)

Alles schön und gut, aber wie stelle ich das jetzt in der Koordinatendarstellung dar?   Einfach $$\begin{pmatrix}3\\1\\0\\-1\end{pmatrix}\cdot \begin{pmatrix}t^3\\t^2\\t^1\\t^0\end{pmatrix}=3t^2+t^2-1$$

schreiben, oder wäre das zu einfach? xD

Avatar von 2,1 k

1 Antwort

+2 Daumen
 
Beste Antwort

Das ist denke ich sogar noch einfacher. Einfach nur der Vektor [3, 1, 0, -1].

Avatar von 477 k 🚀

\(                 \)

Wenn außerdem \(\mathcal{B} = (1, t + 1, t^2 + t + 1, t^3 + t^2 + t + 1)\) und die lineare Abbildung \(f:V\to V\) (mit \(f\) bildet jedes Polynom aus \(V\) auf seine Ableitung ab) gegeben wären, wie könnte ich dann \(M^\mathcal{A}_\mathcal{A}(f)\) und \(M_\mathcal{B}^\mathcal{A}(f)\) berechnen?

Ein anderes Problem?

Stell deine Frage

Willkommen bei der Mathelounge! Stell deine Frage einfach und kostenlos

x
Made by a lovely community